LSAT and Law School Admissions Forum

Get expert LSAT preparation and law school admissions advice from PowerScore Test Preparation.

User avatar
 Dave Killoran
PowerScore Staff
  • PowerScore Staff
  • Posts: 5848
  • Joined: Mar 25, 2011
|
#88278
Complete Question Explanation
(The complete setup for this game can be found here: lsat/viewtopic.php?f=175&p=88237#p88237)

The correct answer choice is (D).

This is the first question of the game that forces you to combine two or more of the rules (in part, this justifies the decision to not show all of the possible combinations during the setup of the game, and to wait until a question forces you to do so). This is also the first legitimately challenging question of the game.

Specifically, the question stem stipulates that J :longline: M. J appears in the second rule, and M appears in the first rule, and so those two rules must be connected first. The third rule can then be connected, through P and G.

Thus, when combining the rules, you must look at both options of rule #2 and impose the J :longline: M relationship as you connect the first rule. Let’s start by looking at both options of rule #2:

Option #1. When the M :longline: L portion of the first rule is combined with this option and the J :longline: M condition in the question stem, the resulting chain is:
G2-Q9-d1.png
From the first rule, P must be presented earlier than M, leading to the following diagram:
G2-Q9-d2.png
V (from the third rule) must be presented before or after both P and G. This effectively allows V great latitude (but not complete freedom, as V could not be presented second or fifth, for example).
Option #2. When the first rule is combined with this option and the condition in the question stem, the following diagram results:
G2-Q9-d3.png
V (from the third rule) must be presented before or after both P and G. With the above configuration, V is pushed either to the front of or the back of the chain.
With the information above, we can attack the questions.

Answer choice (A) can occur because J can be presented second in either of the two options. For example, under option #2 in the hypothetical P-J-M-L-G-V. Thus, this answer choice is incorrect.

Answer choice (B) can occur because P can be presented third in either of the two options. For example, under option #2 in the hypothetical V-J-P-M-L-G. Thus, this answer choice is incorrect.

Answer choice (C) can occur because P can be presented fourth in the first option (but not the second option). For example, under option #1 in the hypothetical V-G-J-P-M-L. Thus, this answer choice is incorrect.

Answer choice (D) cannot occur because in option #1 J is followed by at least M and L, so the latest that J can present is fourth. In option #2, J is followed by at least M, L, and G, and so the latest that J can present is third. Thus, J can never present fifth, and answer choice (D) is correct.

Answer choice (E) can occur because as shown in the hypothetical mentioned in answer choice (B)—V-J-P-M-L-G—L can present fifth, and thus this answer choice is incorrect.
 bethavedon
  • Posts: 7
  • Joined: Jun 20, 2016
|
#28582
Hi,

I came to the correct answer for this, but I think in a more time consuming way than perhaps necessary. What would be the way you'd suggest attacking this question?

Thanks in advance!
 Clay Cooper
PowerScore Staff
  • PowerScore Staff
  • Posts: 241
  • Joined: Jul 03, 2015
|
#28673
Hi Beth,

Thanks for your question!

I think the fastest way to approach this question is this: write down the local rule (J is before M) and then incorporate it into the sequencing rule we already have, which contains P, M, and L. Then I think we should evaluate what we are being asked here: each of the following could be true EXCEPT. In other words, we are looking for an answer choice that must be false.

Moreover, it's probable (though by no means certain) that the correct answer choice will play off of our added rule here - that J must be before M. So, I would think of my original rules and the added rule as my tools to prove an answer choice wrong, and take a peek at the answer choices. I'd start with any that mention J or M, since they are both in the added rule.

That gives us A and D, which both mention J, as favorites. A quick look at A doesn't seem to reveal any reason why it might be wrong - but then, when I look at D, I see that J can never be fifth, because, according to our added rule here (which we looked at first thing), J has at least two variables that must come after it. So, it could never be fifth in this six-variable game.

I hope that helps!
User avatar
 amb8ds
  • Posts: 3
  • Joined: Aug 03, 2022
|
#96626
Why would one need to consider the second option if L is the sixth variable? Wouldn't it be impossible for G to be after L and J since L is last?
User avatar
 atierney
PowerScore Staff
  • PowerScore Staff
  • Posts: 215
  • Joined: Jul 06, 2021
|
#96641
Yes, that is correct. And this is good general point for the logic games. Always look and see if there is a quick answer rather than testing other more plausible answers in an "all of the following EXCEPT" scenario. Here, D is the obvious answer because L has to go after M, thus M can't be last, meaning J can't be 5th in this scenario.

Let me know if you have any questions.

Get the most out of your LSAT Prep Plus subscription.

Analyze and track your performance with our Testing and Analytics Package.